Đến nội dung

Hình ảnh

Topic về bất đẳng thức

* * * * * 16 Bình chọn

  • Please log in to reply
Chủ đề này có 206 trả lời

#1
wallunint

wallunint

    Thượng sĩ

  • Thành viên
  • 273 Bài viết

Topic về bất đẳng thức


1) Lời nói đầu :
Bất đẳng thức là một chủ đề khá thú vị trên các diển đàn hiện nay với một số lượng bài viết khá lớn. Nhưng ở VMF, số bài viết còn chưa nhiều và còn quá loảng, chủ yếu tập trung ở forum toán cấp 2 thôi :D Mình lấp topic này mong các bạn nhiệt tình tham gia ^_^ Qua topic này, mong rằng có thể cùng các bạn thảo luận và khám phá nhiều bất đẳng thức mới.

2) Quy định post bài :
+ Chỉ được post 2 bài 1 lần. Giải xong mới được post tiếp để tránh hiện tượng Spam và loãng topic :D Nếu bài khó quá thì để lại từ từ giải, chuyển qua bài khác.
+ Không Spam
+ Bài giải phải đầy đủ các bước ( nói tóm tắt cũng đc).
+ Dùng từ ngữ đúng theo ngữ pháp Tiếng Việt. Và phải dùng đúng latex.
+ Bài viết vi phạm các quy định trên thì sẽ bị xóa không thương tiếc ^_^


Mở mình xin mở đầu topic bằng 2 bài toán sau ^_^

Bài 1: Cho các số thực không âm $a,b,c$ sao cho không có 2 số nào đồng thời bằng $0$. CMR:

$\sqrt {\dfrac{{{a^3}}}{{{a^2} + ab + {b^2}}}} + \sqrt {\dfrac{{{b^3}}}{{{b^2} + bc + {c^2}}}} + \sqrt {\dfrac{{{c^3}}}{{{c^2} + ac + {a^2}}}} \geqslant \dfrac{{\sqrt a + \sqrt b + \sqrt c }}{{\sqrt 3 }}$


Bài 2: Cho các số thực không âm $a,b,c$ sao cho không có 2 số nào đồng thời bằng $0$. CMR:

$\sqrt {\dfrac{{{a^3}}}{{{a^3} + {{\left( {b + c} \right)}^3}}}} + \sqrt {\dfrac{{{b^3}}}{{{b^3} + {{\left( {a + c} \right)}^3}}}} + \sqrt {\dfrac{{{c^3}}}{{{c^3} + {{\left( {a + b} \right)}^3}}}} \geqslant 1$


Bài viết đã được chỉnh sửa nội dung bởi dark templar: 21-05-2011 - 20:10
Đánh số thứ tự cho bài

Vì cuộc sống luôn thay màu .... !!!


#2
h.vuong_pdl

h.vuong_pdl

    Thượng úy

  • Hiệp sỹ
  • 1031 Bài viết
Nhận xét: Bài 2: mình vẫn nhớ một cách giải bằng cách dùng đáng giá của anh PKH giới thiệu trong cuốn sáng tạo BDT như sau:

$\sqrt{\dfrac{a^3}{(a^3+(b+c)^3}} \ge \dfrac{a^2}{a^2+b^2+c^2}$

làm tương tự r�#8220;i cộng lại theo vế ta có ngay đpcm!

Lời giải tuy rất hay + gọn gẽ nhưng thật sự không thuyết phục lắm :D

Mình cũng thấy cách đánh giá này ở nhiều bài toán, ví dụ IMO năm ???? có đánh giá:

$\dfrac{a}{\sqrt{a^2+8bc}} \ge \dfrac{a^{\dfrac{1}{3}}}{a^{\dfrac{1}{3}}+b^{\dfrac{1}{3}}+c^{\dfrac{1}{3}}}$

Như vậy ta có thể tìm ra con số $2$ hay $\dfrac{1}{3}$ như trên như thế nào, và liệu khi nào thì ta nghĩ đến việc tìm ra cách đánh giá như thế ????

Bài viết đã được chỉnh sửa nội dung bởi h.vuong_pdl: 20-05-2011 - 16:14

rongden_167


#3
dark templar

dark templar

    Kael-Invoker

  • Hiệp sỹ
  • 3788 Bài viết

2) Cho các số thực không âm $a,b,c$ sao cho không có 2 số nào đồng thời bằng $0$. CMR:

$\sqrt {\dfrac{{{a^3}}}{{{a^3} + {{\left( {b + c} \right)}^3}}}} + \sqrt {\dfrac{{{b^3}}}{{{b^3} + {{\left( {a + c} \right)}^3}}}} + \sqrt {\dfrac{{{c^3}}}{{{c^3} + {{\left( {a + b} \right)}^3}}}} \geqslant 1$[/size][/font]

Vui vui tí vậy :D
Có 2 cách chém bài này:
Cách 1: Chứng minh $\sqrt{\dfrac{a^3}{a^3+(b+c)^3}} \ge \dfrac{a^2}{a^2+b^2+c^2}$
Cách 2: Do tính thuần nhất của BĐT nên chuẩn hóa $a+b+c=1$.Khi đó chứng minh hàm $f:[0;+ \infty) \to [0;1):f(x)=\sqrt{\dfrac{a^3}{a^3+(1-a)^3}}$ là hàm lồi trên $[0;1)$,rồi kết hợp với BĐT Jensen,ta sẽ có ngay kết quả ^_^
"Do you still... believe in me ?" Sarah Kerrigan asked Jim Raynor - Starcraft II:Heart Of The Swarm.

#4
wallunint

wallunint

    Thượng sĩ

  • Thành viên
  • 273 Bài viết



2) Cho các số thực không âm $a,b,c$ sao cho không có 2 số nào đ�ồng thời bằng $0$. CMR:

$\sqrt {\dfrac{{{a^3}}}{{{a^3} + {{\left( {b + c} \right)}^3}}}} + \sqrt {\dfrac{{{b^3}}}{{{b^3} + {{\left( {a + c} \right)}^3}}}} + \sqrt {\dfrac{{{c^3}}}{{{c^3} + {{\left( {a + b} \right)}^3}}}} \geqslant 1$


Lần sau anh dark templar và anh h.vuong_pdl giải rõ ra nhá :D Không thì em xóa bài đấy :D
2 cách giải sơ cấp cho bài này như sau :D
Cách 1: Chứng minh $\sqrt {\dfrac{{{a^3}}}{{{a^3} + {{(b + c)}^3}}}} \geqslant \dfrac{{{a^2}}}{{{a^2} + {b^2} + {c^2}}}$

$ \Leftrightarrow 2{a^2}\left( {{b^2} + {c^2}} \right) + {\left( {{b^2} + {c^2}} \right)^2} \geqslant a{\left( {b + c} \right)^3}$

Áp dụng bất đẳng thức Cauchy-Schwarz, ta có:

$2\left( {{b^2} + {c^2}} \right) \geqslant {\left( {b + c} \right)^2} \Leftrightarrow {\text{8}}{\left( {{b^2} + {c^2}} \right)^3} \geqslant {\left( {b + c} \right)^6}$

Tiếp tục áp dụng bất đẳng thức AM-GM, ta có:

${a^2}\left( {{b^2} + {c^2}} \right) + {\left( {{b^2} + {c^2}} \right)^2} \geqslant 2\sqrt {{a^2}{{\left( {{b^2} + {c^2}} \right)}^3}} \geqslant a{\left( {b + c} \right)^3}$

Vậy, phép chứng minh hoàn tất. :D

Cách 2: sử dụng bất đẳng thức AM-GM và viết lại bất đẳng thức như sau :



ps: đi học về post tiếp :D

Bài viết đã được chỉnh sửa nội dung bởi wallunint: 21-05-2011 - 11:45

Vì cuộc sống luôn thay màu .... !!!


#5
Fire-Phoenix

Fire-Phoenix

    Lính mới

  • Thành viên
  • 1 Bài viết
Bài 3:Cho a, b, c là các số thực dương. CMR:

$\dfrac{1}{{a + b + c}} + \dfrac{1}{{{a^2} + {b^2} + {c^2}}} + \dfrac{1}{{{a^3} + {b^3} + {c^3}}} + \dfrac{{{a^6} + {{(b + c)}^6}}}{4} \geqslant 2$



Bài viết đã được chỉnh sửa nội dung bởi dark templar: 21-05-2011 - 20:11
Đánh số thứ tự bài


#6
truclamyentu

truclamyentu

    Sĩ quan

  • Thành viên
  • 333 Bài viết
Bài 4:chứng minh với mọi số thực không âm a,b,c ta có

$\dfrac{{b + c}}{{2{a^2} + bc}} + \dfrac{{c + a}}{{2{b^2} + ac}} + \dfrac{{a + b}}{{2{c^2} + ab}} \ge \dfrac{6}{{a + b + c}}$

Bài viết đã được chỉnh sửa nội dung bởi dark templar: 21-05-2011 - 20:12
Đánh số thứ tự bài


#7
dark templar

dark templar

    Kael-Invoker

  • Hiệp sỹ
  • 3788 Bài viết

chứng minh với mọi số thực không âm a,b,c ta có

$\dfrac{{b + c}}{{2{a^2} + bc}} + \dfrac{{c + a}}{{2{b^2} + ac}} + \dfrac{{a + b}}{{2{c^2} + ab}} \ge \dfrac{6}{{a + b + c}}$

Bài này phải có thêm đk $ab+bc+ca>0$ để đảm bảo ĐKXĐ nữa nha bạn :D
Xem ở đây
P/s:Bài này anh Cẩn giải kinh khủng thật :D.Và cũng đề nghị các mem post bài là hãy đánh số thứ tự cho bài gửi của mình nhằm tiện hơn cho việc theo dõi.Thân.
Bài 5: Cho $n$ số thực $x_{i}(i=\overline{1,n})$ thỏa $x_1^2+x_2^2+...+x_{n}^2=1$.Chứng minh rằng:
$\dfrac{x_1}{1+x_1^2}+\dfrac{x_2}{1+x_1^2+x_2^2}+...+\dfrac{x_{n}}{1+x_1^2+x_2^2+...+x_{n}^2} <\sqrt{\dfrac{n}{2}}$
(Đề thi đề nghị Olympiad 30-4)

Bài 6: Cho $a,b,c \ge 0;ab+bc+ca=3$.Chứng minh rằng:$\dfrac{9}{10} \le \dfrac{1}{a^2+2}+\dfrac{1}{b^2+2}+\dfrac{1}{c^2+2} \le 1$
(Thái Nguyễn Hưng)

Bài viết đã được chỉnh sửa nội dung bởi dark templar: 21-05-2011 - 20:40

"Do you still... believe in me ?" Sarah Kerrigan asked Jim Raynor - Starcraft II:Heart Of The Swarm.

#8
Pirates

Pirates

    Mathematics...

  • Thành viên
  • 642 Bài viết
Góp vui 2 bài...

Bài 7: Cho $a, b, c$ là ba số thực dương tùy ý. Chứng minh rằng:

$\sum\limits_{cyc} \sqrt{\dfrac{b + c}{a}} \ge 2\left(\sum\limits_{cyc} \sqrt{\dfrac{a}{b + c}}\right).\sqrt{1 + \dfrac{(a + b)(b + c)(c + a) - 8abc}{4\sum\limits_{cyc} a(a + b)(a + c)}}.$

Bài 8: Cho $a, b, c > 0$ và $x, y \geq 1$ là các số thực. Chứng minh rằng:

$\sqrt[3]{abc} \leq \sqrt[6]{\dfrac{\left(\sum\limits_{cyc} a^2 b^2 + 2x \sum\limits_{cyc} a^2 bc\right)\left(\sum\limits_{cyc} a^2 + 2y \sum\limits_{cyc} ab\right)}{(3 + 6x)(3 + 6y)}} \leq \dfrac{1}{3} \sum\limits_{cyc} a.$

Bài viết đã được chỉnh sửa nội dung bởi Pirates: 21-05-2011 - 20:36

"God made the integers, all else is the work of men"


#9
dark templar

dark templar

    Kael-Invoker

  • Hiệp sỹ
  • 3788 Bài viết

Bài 8: Cho $a, b, c > 0$ và $x, y \geq 1$ là các số thực. Chứng minh rằng:

$\sqrt[3]{abc} \leq \sqrt[6]{\dfrac{\left(\sum\limits_{cyc} a^2 b^2 + 2x \sum\limits_{cyc} a^2 bc\right)\left(\sum\limits_{cyc} a^2 + 2y \sum\limits_{cyc} ab\right)}{(3 + 6x)(3 + 6y)}} \leq \dfrac{1}{3} \sum\limits_{cyc} a.$

Bài 8:Bài này xài Đạo Hàm giải mệt quá :D.
Sử dụng BĐT AM-GM,ta có:$ \left\{\begin{array}{l}\sum a^2b^2 \ge \sum_{cyc} a^2bc=abc(a+b+c)\\\sum a^2 \ge \sum ab\end{array}\right. $
Như vậy,ta có:$ \sqrt[6]{\dfrac{\left(\sum\limits_{cyc} a^2 b^2 + 2x \sum\limits_{cyc} a^2 bc\right)\left(\sum\limits_{cyc} a^2 + 2y \sum\limits_{cyc} ab\right)}{(3 + 6x)(3 + 6y)}} \ge \sqrt[6]{\dfrac{(1+2x)(1+2y)abc(a+b+c)\sum ab}{9(1+2x)(1+2y)}}$
$=\sqrt[6]{\dfrac{abc(a+b+c)(ab+bc+ca)}{9}} \ge \sqrt[6]{a^2b^2c^2}=\sqrt[3]{abc}$
Việc còn lại của ta là chứng minh $ \sqrt[6]{\dfrac{\left(\sum\limits_{cyc} a^2 b^2 + 2x \sum\limits_{cyc} a^2 bc\right)\left(\sum\limits_{cyc} a^2 + 2y \sum\limits_{cyc} ab\right)}{(3 + 6x)(3 + 6y)}} \le \dfrac{1}{3}\sum a$
Xét hàm số $f(x)= \sqrt[6]{\dfrac{\left(\sum\limits_{cyc} a^2 b^2 + 2x \sum\limits_{cyc} a^2 bc\right)\left(\sum\limits_{cyc} a^2 + 2y \sum\limits_{cyc} ab\right)}{(3 + 6x)(3 + 6y)}}(x \ge 1)$
$f'(x)=\sqrt[6]{\dfrac{\sum a^2 +2y\sum ab}{3(3+6y)}}.\dfrac{2abc(a+b+c)(1+2x)-2\left[\sum a^2b^2 +2xabc(a+b+c) \right]}{6(1+2x)^2.\sqrt[6]{\left(\dfrac{\sum a^2b^2 +2xabc(a+b+c)}{1+2x} \right)^5}}$
$=\sqrt[6]{\dfrac{\sum a^2 +2y\sum ab}{3(3+6y)}}.\dfrac{abc(a+b+c)-\sum a^2b^2}{3(1+2x)^2.\sqrt[6]{\left(\dfrac{\sum a^2b^2 +2xabc(a+b+c)}{1+2x} \right)^5}} \le 0,\forall x \in [1;+ \infty)$
$ \Rightarrow f(x) \le f(1)=\sqrt[6]{\dfrac{\left(\sum\limits_{cyc} a^2 b^2 + 2 \sum\limits_{cyc} a^2 bc\right)\left(\sum\limits_{cyc} a^2 + 2y \sum\limits_{cyc} ab\right)}{9(3 + 6y)}}$
Làm tương tự,ta cũng chứng minh được:
$\sqrt[6]{\dfrac{\left(\sum\limits_{cyc} a^2 b^2 + 2 \sum\limits_{cyc} a^2 bc\right)\left(\sum\limits_{cyc} a^2 + 2y \sum\limits_{cyc} ab\right)}{9(3 + 6y)}} \le \sqrt[6]{\dfrac{\left(\sum\limits_{cyc} a^2 b^2 + 2 \sum\limits_{cyc} a^2 bc\right)\left(\sum\limits_{cyc} a^2 + 2 \sum\limits_{cyc} ab\right)}{9.9}}=\sqrt[3]{\dfrac{(a+b+c)(ab+bc+ca)}{9}}$
Như vậy,ta chỉ cần chứng minh:$\sqrt[3]{\dfrac{(a+b+c)(ab+bc+ca)}{9}} \le \dfrac{a+b+c}{3} \Leftrightarrow 3\sum ab \le (a+b+c)^2$
$ \Leftrightarrow (a-b)^2+(b-c)^2+(c-a)^2 \ge 0$(luôn đúng).
Vậy ta có đpcm.Đẳng thức xaỷ ra khi $ \left\{\begin{array}{l}x=y=1\\a=b=c\end{array}\right.$
-------------------------------------------------------------------------------------------------------------------------------------
P/s:Anh Pirates còn cách nào khác ngắn gọn hơn không ạ :D.Còn bài trên của anh,em nhìn vô thấy ngán quá,để xem hôm nay có chém được không :D
Còn bài của bạn Fire-Phoenix thì xem ở đây

Bài viết đã được chỉnh sửa nội dung bởi dark templar: 22-05-2011 - 10:24

"Do you still... believe in me ?" Sarah Kerrigan asked Jim Raynor - Starcraft II:Heart Of The Swarm.

#10
h.vuong_pdl

h.vuong_pdl

    Thượng úy

  • Hiệp sỹ
  • 1031 Bài viết
Bài 6:

Đặt $x = bc, y = ca, z = ab$ thì ta có $x+y+z = 3$ và cần chứng minh:

$\sum{\dfrac{x}{yz+2x}} \le 1 \Leftrightarrow \sum{\dfrac{yz}{yz+2x}} \ge 1$

Áp dụng BDT Cauchy-Schwarz, ta có:

$\textup{VT} = \sum{\dfrac{y^2z^2}{y^2z^2+2xyz}} \ge \dfrac{(xy+yz+zx)^2}{\sum{x^2y^2} + 6xyz} = \dfrac{\sum{x^2y^2} + 6xyz}{\sum{y^2z^2}+6xyz} = 1$

Vậy ta có đpcm!

rongden_167


#11
traitimbangtuyet

traitimbangtuyet

    Lính mới

  • Thành viên
  • 8 Bài viết
ai giải gúp tớ với :
biết $x^2+y^2=52$ . Tính giá trị lớn nhất của biểu thức $A=3x+2y$

Bài viết đã được chỉnh sửa nội dung bởi Phạm Quang Toàn: 22-05-2011 - 11:26


#12
wallunint

wallunint

    Thượng sĩ

  • Thành viên
  • 273 Bài viết

Bài 7: Cho $a, b, c$ là ba số thực dương tùy ý. Chứng minh rằng:

$\sum\limits_{cyc} \sqrt{\dfrac{b + c}{a}} \ge 2\left(\sum\limits_{cyc} \sqrt{\dfrac{a}{b + c}}\right).\sqrt{1 + \dfrac{(a + b)(b + c)(c + a) - 8abc}{4\sum\limits_{cyc} a(a + b)(a + c)}}.$


Các bạn xem lời giải của mình ở đây :D

Solution

Bài viết đã được chỉnh sửa nội dung bởi wallunint: 22-05-2011 - 13:24

Vì cuộc sống luôn thay màu .... !!!


#13
anhtuanDQH

anhtuanDQH

    Thượng sĩ

  • Thành viên
  • 236 Bài viết

ai giải gúp tớ với :
biết $x^2+y^2=52$ . Tính giá trị lớn nhất của biểu thức $A=3x+2y$


Áp dụng bất đẳng thức Cauchy-Scgwarz, ta có:

$\ (x^2+y^2)( 9 +4 ) \geq (3x+2y )^2 \Rightarrow DPCM $

P/s: Ðây là môt BÐT đơn giản, vì vậy ban nên tự nghĩ trước khi post nha. .
Còn về bài làm anh Lê Xuân Trường Gjang làm sai , bài ko cho là sô dương làm sao A lai rút được x như vậy. :D



wallunint ps: nhớ gõ đúng Tiếng Việt :D

Bài viết đã được chỉnh sửa nội dung bởi wallunint: 22-05-2011 - 13:04
Lỗi Tiếng Việt

Xăng có thể cạn, lốp có thể mòn..xong số máy số khung thì không bao giờ thay đổi

NGUYỄN ANH TUẤN - CHỦ TỊCH HIỆP HỘI
Hình đã gửi


#14
wallunint

wallunint

    Thượng sĩ

  • Thành viên
  • 273 Bài viết
Tiếp tục nào :D
Bài 9: Cho các số thực không âm $a,b,c$ thỏa mãn$a+b+c=2$. CMR:

$\dfrac{{bc}}{{{a^2} + 1}} + \dfrac{{ac}}{{{b^2} + 1}} + \dfrac{{ab}}{{{c^2} + 1}} \leqslant 1$


*Nhận xét : Bài này có khá nhiều cách giải rất trâu bò
Các bạn có thể tìm đc bao nhiêu cách giải cho bài này ??? :D


Bài 2: Cho các số thực không âm $a,b,c$ sao cho không có 2 số nào đồng thời bằng $0$. CMR:

$\sqrt {\dfrac{{{a^3}}}{{{a^3} + {{\left( {b + c} \right)}^3}}}} + \sqrt {\dfrac{{{b^3}}}{{{b^3} + {{\left( {a + c} \right)}^3}}}} + \sqrt {\dfrac{{{c^3}}}{{{c^3} + {{\left( {a + b} \right)}^3}}}} \geqslant 1$

Các bạn nhanh chóng giải bài này đi :D
Gợi ý : Chỉ sử dụng bất đẳng thức $AM-GM$ $(Cauchy)$

Đừng post bài nữa nhá :D để giải xong mấy bài trên rồi post tiếp nha :D

Vì cuộc sống luôn thay màu .... !!!


#15
dark templar

dark templar

    Kael-Invoker

  • Hiệp sỹ
  • 3788 Bài viết
Đưa tiếp 1 bài lên vậy :(
Bài 10: Cho $a,b,c>0;a+b+c=3$.Chứng minh rằng:
$\dfrac{bc}{\sqrt{a^2+3}}+\dfrac{ac}{\sqrt{b^2+3}}+\dfrac{ab}{\sqrt{c^2+3}} \le \dfrac{3}{2}$
"Do you still... believe in me ?" Sarah Kerrigan asked Jim Raynor - Starcraft II:Heart Of The Swarm.

#16
Giang1994

Giang1994

    C'est la vie

  • Thành viên
  • 249 Bài viết
nhận lời mời dự tiệc mà giờ mới vô được
bài 11
cho các số thực dương a, b, c tm a+b+c=1
CmR:
$\sqrt{a+\dfrac{(b-c)^2}{4}}+\sqrt{b}+\sqrt{c} \leq \sqrt{3} $

olympic toán nữ sinh TQ

--------
bài này có vẻ cũ nhưng hay :(

Don't let people know what you think


#17
kiemsac124

kiemsac124

    Lính mới

  • Thành viên
  • 1 Bài viết
:ukliam2: :ukliam2: đây là bài thi thử đại học khối A dành cho khối 10 tham dự
Cho $a,b,c \geq 0; a + b + c \leq 3$.Chứng minh rằng:
$\dfrac{1}{1+a} + \dfrac{1}{1+b} + \dfrac{1}{1+c} \geq \dfrac{a}{1+ a^{2} } + \dfrac{b}{1+b^{2} } + \dfrac{c}{1+ c^{2} }$

Hình gửi kèm

  • gif.latex.gif

Bài viết đã được chỉnh sửa nội dung bởi kiemsac124: 31-10-2011 - 17:28
Cần gõ Latex trong bài viết


#18
SLNA

SLNA

    Bảo Duyên

  • Thành viên
  • 77 Bài viết

Đưa tiếp 1 bài lên vậy :(
Bài 10: Cho $a,b,c>0;a+b+c=3$.Chứng minh rằng:
$\dfrac{bc}{\sqrt{a^2+3}}+\dfrac{ac}{\sqrt{b^2+3}}+\dfrac{ab}{\sqrt{c^2+3}} \le \dfrac{3}{2}$

Ta có $a+b+c=3\geq ab+bc+ca$. Nên
$\dfrac{bc}{\sqrt{a^2+3}}+\dfrac{ac}{\sqrt{b^2+3}}+\dfrac{ab}{\sqrt{c^2+3}} \le \sum \dfrac{bc}{\sqrt{(a+c)(a+b)}}\leq \sum \dfrac{1}{2}\left ( \dfrac{bc}{a+b}+\dfrac{ac}{a+b} \right )=\dfrac{3}{2}$

#19
dark templar

dark templar

    Kael-Invoker

  • Hiệp sỹ
  • 3788 Bài viết

giúp mình bài này với, dạo này tư duy kém quá, đây là bài thi thử đại học khối A dành cho khối 10 tham dự
Cho $a,b,c \geq 0; a + b + c \leq 3$.Chứng minh rằng:
$\dfrac{1}{1+a} + \dfrac{1}{1+b} + \dfrac{1}{1+c} \geq \dfrac{a}{1+ a^{2} } + \dfrac{b}{1+b^{2} } + \dfrac{c}{1+ c^{2} }$

Sử dụng BĐT Cauchy-Schwarzt+AM-GM,ta có:
$VT \ge \dfrac{9}{3+\sum a} \ge \dfrac{9}{3+3}=\dfrac{3}{2} \ge \sum \dfrac{a}{1+a^2}=VP$
"Do you still... believe in me ?" Sarah Kerrigan asked Jim Raynor - Starcraft II:Heart Of The Swarm.

#20
anh qua

anh qua

    Sĩ quan

  • Hiệp sỹ
  • 476 Bài viết
Bài 11:Cho a,b,c là các số thực dương thỏa mãn $abc=1.$ CM:
$\dfrac{(a^4b^4+b^4c^4+c^4a^4)}{a^2+b^2+c^2}+\dfrac{8}{(ab+a)(bc+ba)(ca+b)}\geq 2$
bài này do mình tự chế, anh cường và các bạn chém nhé.
Bài 12: cho $a,b,c$ là các số thực thỏa mãn:
$a+b+c=m^2; a^2+b^2+c^2=n^2;3n^2 \geq m^2.$
Tìm min, max :
$a^2b+b^2c+c^2a$
Bài 13:Cho $a,b,c,x,y,z$ là các số thực dương thỏa mãn:
$x\geq y \geq z;a \leq x ;a^2+b^2 \leq x^2+y^2 ; a^3+b^3+c^3 \leq x^3+y^3+z^3.$
Chứng minh :$ a^6+b^6+c^6 \leq x^6+y^6+z^6.$
(thi HSG tổng hợp)

Bài viết đã được chỉnh sửa nội dung bởi dark templar: 23-05-2011 - 21:10
Đánh số thứ tự bài

Give me some sunshine
Give me some rain
Give me another chance
I wanna grow up once again




0 người đang xem chủ đề

0 thành viên, 0 khách, 0 thành viên ẩn danh